User avatar
 
ohthatpatrick
Thanks Received: 3806
Atticus Finch
Atticus Finch
 
Posts: 4661
Joined: April 01st, 2011
 
 
 

Q12 - If an activity significantly reduces lower back

by ohthatpatrick Thu Oct 26, 2017 1:27 am

Question Type:
Necessary Assumption

Stimulus Breakdown:
Conclusion: Docs treating lower back patients should be prepared to discuss yoga.
Evidence: If an activity significantly reduces back pain, docs should be prepared to discuss it. A recent study found that yoga reduced back pain as much as did stretching classes with a PT.

Answer Anticipation:
To trigger the rule that says "doc should be prepared to talk about X", we have to establish that "X significantly reduces back pain".

Have we established that fact about yoga? No, we've just said that it reduces pain as much as stretching classes do. So we have to get from "yoga reduces back pain as much as stretching classes do" to "yoga significantly reduces back pain".

That means that we're missing the idea that "stretching classes significantly reduce chronic lower back pain".

Correct Answer:
B

Answer Choice Analysis:
(A) TOO STRONG: there's NOTHING that reduces back pain more? We only need to know it reduces back pain significantly. We don't need stretching classes to be the NUMBER ONE reliever.

(B) YES, we need this. If it were negated, then stretching classes don't signifcantly reduce back pain. Since yoga has an equal reduction in back pain to stretching classes, then we would know that yoga doesn't signifcantly reduce back pain, which means we can't apply the rule in the first sentence. Since negating this answer would badly weaken the argument, we know it's a needed assumption.

(C) TOO STRONG: "few". The author's argument doesn't hinge on yoga and stretching being two of a very limited set of options. Even if there are 50 options for relieving back pain, the argument would still hinge on the missing knowledge of whether stretching classes provide a significant reduction in back pain.

(D) TOO STRONG: "No". The author doesn't need to make any assumptions about ALL previous studies. The author definitely assumes the recent study is trustworthy enough to use as evidence, but that's it.

(E) Whether many doctors do / don't discuss stretching with their patients is a separate issue from what the author is discussing: whether doctors should / shouldn't be prepared to discuss it.

Takeaway/Pattern: This is a carbon copy of a previous test's Necessary Assumption question about using hatha yoga as a way to quit smoking. It's from test 37, check it out: https://www.manhattanprep.com/lsat/foru ... t2022.html

When students ask me, "Is there any point in doing older tests?", I say "Yes!" :)

#officialexplanation